Sunteți pe pagina 1din 33

Problem 3-30: Contribution margin, decision making.

Given:
McCarthy Men's Clothing's revenues and cost data for 2014 are as
follows:

Revenues $500,000
Cost of Goods Sold (50% of Sales) 0.50 250,000
Gross Margin $250,000 50%
Operating Costs:
Salaries (fixed) $160,000
Sales Commission (11% of Sales) 55,000
Depreciation of Equipment and Fixtures 15,000
Store Rent ($4,000 per month) 48,000
Other Operating Costs 40,000 318,000
Operating Income (Loss) ($68,000)

Mr. McCarthy, the owner of the store, is unhappy with the operating results.
An analysis of "Other Operating Costs" reveals that it includes $35,000 of
variable costs, which vary with sales volume, and $5,000 of fixed costs.

1. Compute the contribution margin of McCarthy Men's Clothing.

Sales $500,000
Less: Variable Costs
Cost of Goods Sold 0.500 $250,000
Sales Commissions 0.110 55,000
Other Operating Costs 0.070 35,000 340,000 0.680
Total Contribution Margin 0.320 $160,000 0.320
% of Sales $160,000
2. Compute the C/M %.

Total contribution margin $160,000 0.320 32.0%


Sales Revenue $500,000

3. Mr. McCarthy estimates he can increase revenues by 20% by incurring additional


advertising costs of $12,000. Calculate the impact of the additional advertising
costs on operating income.

Short Way:
Sales increase $100,000
Contribution margin ratio 0.32
Positive change in contribution margin $32,000
Less: increase in fixed advertising cost (12,000)
Positive change in operating income $20,000

Long Way:
Sales $600,000
Cost of Goods Sold 300,000 $300,000
Gross Margin $300,000
Operating Costs
Salaries (fixed) $160,000
Sales Commissions 66,000 66,000
Depreciation 15,000
Store Rent 48,000
Other Operating Costs
Fixed Portion 5,000
Variable Portion 42,000 42,000
Advertising 12,000
Total Operating Costs $348,000
New Operating Income ($48,000)
Less: Old Operating Income (68,000)
Positive change in operating income $20,000

4. What other actions can Mr. McCarthy take to improve operating income?

To improve operating income, Mr. McCarthy must find ways to decrease variable
costs, decrease fixed costs, or increase selling prices.
Problem 3-24: CVP analysis, margin of safety

Given data: Suppose Lattin Corp.'s breakeven point is revenues of $1,500,000. Fixed
costs are $720,000.

Fixed costs $720,000


Breakeven dollars $1,500,000

1. Compute the contribution margin percentage.

Sales = TVC + TFC + NOI BE $ = TFC/CM% $1,500,000 BE Sales


At breakeven: Let X = CM % (720,000) TFC
Sales = TVC + TFC $1,500,000 = $720,000 / X $780,000 TVC
Sales - TVC = TFC Multiply both sides of the equation by X $720,000 TCM
TCM = TFC $1,500,000 X = $720,000 48% CM %
(Sales)(CM%) = TFC X = $720,000 / $1,500,000 52% VC %
BE Sales = TFC/CM% X= 48% 100% CM% +VC%

2. Compute the selling price if variable costs are $13 per unit.

CM % = 48%
Therefore, VC % = 52%
VC% = TVC/Sales = [(VC/Unit) X (Units)] / [SP X Units] = (VC/Unit) / SP
.52 = $13 / SP
.52(SP) = $13 Alt. Solution 60,000 BE Units
SP = $13 / .52 = $25.00 $25

3. Suppose 90,000 units are sold. Compute the margin of safety in units and dollars (M/S).

BE $ = $1,500,000
BE units = $1,500,000 / $25
BE units = 60,000

MS = Actual Sales Level in Units - BE units


MS = 90,000-60,000 = 30,000 margin of safety in units
X $25 selling price
$750,000 margin of safety in sales $
33.333% margin of safety as a %
33.333% margin of safety as a %

4. What does this tell you about the risk of Lattin operating at a loss? What are the most likely
reasons for this risk to increase?

The risk of operating at a loss is low. Sales would need to decrease by 30,000 units (M/S) before
Lattin Corp. will operate at a loss.

The most likely reasons for this risk to increase are:


1. Increase in competition,
2. Weakness in the economy,
3. Bad management
Problem 3-25: Operating leverage.

Given data: Carmel Rugs is holding a two-week carpet sale at


Jean's Club, a local warehouse store. Carmel Rugs plans to sell carpets
for $1,000 each. The company will purchase the carpets from a local
distributor for $400 each, with the priviledge of returning any unsold units
for a full refund. Jean's Club has offered Carmel Rugs two payment
alternatives for the use of space.

Option 1: Floor space charge -- fixed payment of $17,400


Option 2: Floor space charge -- 20% of revenues ?

Assume that Carmel Rugs will incur no other costs.


Selling price expected per carpet = $1,000
Variable cost per carpet = $400

1. Compute the breakeven point for each option.

Option 1: Floor space charge -- fixed payment of $17,400.

Sales = TVC + TFC + P


(SP)(Units) = (VC/Unit)(Units) + $17,400 + 0
$1,000(X) - $400(X) = $17,400
$600(X) = $17,400
X = $17,400/$600 = 29.00 29

Option 2: Floor space charge -- 20% of revenues

Sales = TVC + TFC + P


(SP)(Units) = (VC/Unit)(Units) + $0 + $0
(SP)(Units) = (VC/Unit)(Units)
$1,000(X) = $400(X) + (.20)($1,000)(X)
$1,000(X) = $400(X) + $200(X)
$1,000(X) = $600(X)
$1,000(X) - $600(X) = $0
$400(X) = $0
X=0

2. At what level of revenue will Carmel Rugs earn the same


operating income under either option?
(Point of Indifference)

Option 1: Floor space charge -- fixed payment of $17,400.


Sales(1) = TVC(1) + TFC(1) + P(1)

Option 2: Floor space charge -- 20% of revenues


Sales(2) = TVC(2) + TFC(2) + P(2)

Since at the point of indifference P(1) = P(2), then


Option 1: $1,000(X) - $400(X) - $17,400 = P(1) = P
$600(X) - $17,400 = P

Option 2: $1,000(X) - ($400+$200)(X) - $0 = P(2) = P


$400(X) = P

From Geometry: Things equal to the same thing


are equal to each other.

Therefore,
$600(X) - $17,400 = $400(X)
$200(X) = $17,400
X = $17,400/$200
X = 87 87 Carpets
Sales = 87 X $1,000 = $87,000 Sales Dollars

For what range of unit sales will Carmel Rugs prefer


Option 1? Option 2?
Proof
Option 1 Option 2
Range Preferred Units Units
From To Option 1 or 200 1 or 200
0 87 2 ($16,800) $400 1 unit falls in range of 0-87
87 87+ 1 $102,600 $80,000 200 units fall in range of more than 87 units

3. Calculate the degree of operating leverage at a sales level of


87 units for each option.

Operating leverage = Contribution Margin / Operating Income

Option 1 Option 2
Units Units
87 87
Contribution Margin $52,200 $34,800
Operating Income $34,800 $34,800
Operating Leverage 1.50 1.00

4. Briefly explain and interpret your answers to requirement 3

The degree of operating leverage at a given level of sales helps


managers calculate the effect of fluctuations in sales on operating
income.

% Change in Sales X Operating Leverage = % Change in OI

Therefore, if the % change in sales is 10%, then the % change in OI is

Option % Change Operating % Change


Number in Sales Leverage in OI
1 10% 1.50 15%
2 10% 1.00 10%
ge of more than 87 units
Problem 3-22: CVP analysis, income taxes.

Given: The Swift Meal has two restaurants that are open 24-hours a day.
Fixed costs for the two restaurants together total $456,000 per year.
Service varies from a cup of coffee to full meals. The average sales check
per customer is $9.50. The average cost of food and other variable costs for
each customer is $3.80. The income tax rate is 30%. Target NI is $159,600.

Fixed costs $456,000


Average variable cost $3.80
Average sales check $9.50
Income tax rate 30%
Target net income $159,600

1. Compute the revenues needed to obtain the target net income.

Since:
Target Sales - Target Variable Costs - Target Fixed Costs -Target Taxes = Target NI
And:
Target NI = Target OI - Target Taxes
Then:
Target Sales - Target Variable Costs - Target Fixed Costs -Target Taxes = Target OI - Target Taxes
Since both sides of the equal sign include "-Target Taxes", then we arrive at our basic BE equation of
Target Sales - Target Variable Costs - Target Fixed Costs = Target OI
Combining terms yields:
Target CM - Target Fixed Costs = Target OI
Target CM - $456,000 = Target OI

Calculate Target OI:


Target Taxes = Target Tax Rate X Target OI
Target Taxes = (.3) X Target OI = (.3)Target OI

Target NI = Target OI - Target Taxes


$159,600 = Target OI - (.3) Target OI
$159,600 = (.7) Target OI
Target OI = $159,600 / .7 = $228,000

Target CM - $456,000 = Target OI


Target CM - $456,000 = $228,000
Target CM = $456,000 + $228,000
Target CM = $684,000 $684,000

Target CM = CM% X Target Sales


$684,000 = CM% X Target Sales

Calculate CM%:
VC % = VC/Unit / SP/Unit
VC % = $3.80 / $9.50 = 40%
CM% = 1 - VC% = 1 - .4 = 60% 60%
$684,000 = CM% X Target Sales
$684,000 = (.60) X Target Sales
$684,000/.60 = Target sales = $1,140,000

Shorter Way
Target Sales - Target Variable Costs - Target Fixed Costs -Target Taxes = Target NI
$9.5X - $3.8X - $456,000 - Target Taxes = $159,600

Target Taxes = Target Tax Rate X Target NOI


$9.5X - $3.8X - $456,000 - (.3)(Target NOI) = $159,600

Target NI = Target NOI - Target Taxes


$159,600 = Target NOI - .3(Target NOI)
$159,600 = .7(Target NOI)
Target NOI = $159,600 / .7 = $228,000
Then
$9.5X - $3.8 X - $456,000 - (.3)(Target NOI) = $159,600
$9.5X - $3.8 X - $456,000 - (.3)($228,000) = $159,600
$9.5X - $3.8 X - $456,000 - $68,400 = $159,600
$5.70X - $456,000 - $68,400 = $159,600
$5.70X - $524,400 = $159,600
$5.70X = $159,600 + $524,400 = $684,000
$5.70X = $159,600 + $524,400 = $684,000
X = $684,000/$5.70 = 120,000 sales units
Selling price per unit $9.50
Sales Revenue $1,140,000

2. How many customers are needed to B/E? To earn NI of $159,600?

Customers necessary to break even:


Let X = BE units
Sales - TVC - TFC = 0
$9.5(X) - $3.80(X) - $456,000 = 0
$5.7(X) = $456,000
X = $456,000 / $5.7 = 80,000 customers needed to BE

Customers necessary to earn NI of $159,600:


From Q1 above: Target Sales = $1,140,000
Target Sales Revenue / SP = Units
Target Sales Revenue / SP = $1,140,000 / $9.5 = 120,000
Proof 120,000
Proof 120,000

3. Calculate the NI if the number of customers is 145,000.

Contribution Margin = $5.7 X 145,000 = $826,500


Less fixed costs (456,000)
Operating Income $370,500
Less taxes at 30% (111,150)
Net Income $259,350
Proof $259,350
OI - Target Taxes
basic BE equation of
Problem 3-36 (CMA adapted)

Given data:
J. T. Brooks, a manufacturer of quality handmade walnut bowls, has
had a steady growth in sales for the past five years. However, increased
competition has led Mr. Brooks, the president, to believe that an aggressive
marketing campaign will be necessary next year to maintain the company's
present growth. To prepare for next year's marketing campaign, the
company's controller has prepared and presented Mr. Brooks with the
following data for the current year, 2014:

Variable cost (per bowl)


Direct materials $3.00
Direct Manufacturing labor 8.00
Variable overhead (manufacturing, marketing, distribution,
and customer services) 7.50
Total variable cost per bowl $18.50

Fixed costs
Manufacturing $20,000
Marketing, distribution, and customer service 194,500
Total fixed costs $214,500

Selling price $35.00


Expected sales
Units 22,000
Dollars $770,000
Income tax rate 40%

1. What is the projected net income for 2014?

Sales -Total Variable Cost - Total Fixed Costs = Operating Income


$770,000 - ($18.50)(22,000) - $214,500 = OI
$770,000 - $407,000 - $214,500 = OI
OI = $148,500 Before tax value

Taxes = Tax Rate X OI = .4 X OI


NI = OI - Taxes = $148,500 - (.4)($148,500)
NI = $148,500 - $59,400 = $89,100

2. What is the breakeven point in units for 2014?

Let X = BE units
Sales - TVC - TFC = 0
$35(X) - $18.50(X) - $214,500 = 0 16.50
$16.50(X) = $214,500
X = $214,500 / $16.50 = 13,000 units

3. Mr. Brooks has set the revenue target for 2015 at a level of $875,000
(or 25,000 bowls). He believes an additional marketing cost of
$16,500 for advertising in 2015, with all other costs remaining
constant, will be necessary to attain the revenue target. What is
the net income for 2015 if the additional $16,500 is spent and the
revenue target is met?

Change in volume 3,000


CM per bowl $16.50 Proof
Change in total CM $49,500 Sales $875,000
Change in fixed costs (16,500) Variable Costs 462,500
Change in operating income $33,000 Contribution $412,500 $412,500
Less change in income taxes (40%) 13,200 Fixed Costs 231,000
Change in NI $19,800 NOI $181,500
Old net income 89,100 Taxes (40%) 72,600
New net income $108,900 Net Income $108,900

4. What is the BE point in revenues for 2015 if the additional $16,500


is spent on advertising?

BE Dollars = FC/CM Ratio


BE Dollars = ($214,500 + $16,500) / ($16.50/$35.00)
BE Dollars = $231,000 0.47142857 $490,000

Alternative solution: extra sales units needed to cover extra $16,500 of advertising
1,000 $490,000

5. If the additional $16,500 is spent, what are the required 2015 revenues
for 2015 net income to equal 2014 net income?

NI for 2014 (22,000 units) = $89,100 See #1 above


OI for 2014 (22,000 units) = $148,500 See #1 above

Required revenues for 2015:


Sales - TVC -TFC = $148,500
(SP)(Units) - (VC/Unit)(Units) - ($214,500 + $16,500) = $148,500
$35X - $18.50X - $231,000 = $148,500
$16.50X = $148,500 + $231,000
X = $379,500/$16.50 = 23,000 units
Sales Dollars = ($35)(23,000) = $805,000

Alternative solution:
OI 2015 = OI 2014
Therefore the change in TCM must = the change in total advertising dollars of $16,500
Since CM per unit = $16.50, then ($16,500/$16.50) = 1,000 additional units must be sold.
Since 22,000 units were sold in 2014, then 22,000 + 1,000 = 23,000 units must be sold in 2015.
Therefore the required revenue for 2015 = 23,000 X $35 = $805,000

6. At a sales level of 25,000 units, what maximum amount can be spent on


advertising if a 2015 net income of $108,450 is desired?

Net income = .6(OI) (Tax rate = 40%).


$108,450 = .6X
X = $108,450/.6
X= $180,750 Operating income

Sales = TVC + TFC + OI


$35(25,000) = $18.50(25,000) + ($214,500 + X) + $180,750
$16.50(25,000) = ($214,500 + X) + $180,750
$412,500 = $395,250 + X
X= $17,250
Problem 3-42 (CMA adapted)

Given data:
Carlisle Engine Company manufactures and sells diesel engines for use in
small farming equipment. For its 2014 budget, Carlisle Engine Company
estimates the following:
Selling price expected per engine = $4,000
Variable cost per engine = $1,000
Annual fixed costs = $4,800,000
Estimated NI $1,200,000 $1,500,000 NOI
Income tax rate expected = 20%

The first quarter income statement, as of March 31, reported that sales were
not meeting expectations. During the first quarter, only 400 units had been
sold at the current price of $4,000. The income statement showed that
variable and fixed costs were as planned, which meant that the 2014 annual
net income projection would not be met unless management took action. A
management committee was formed and presented the following mutually
exclusive alternatives to the president:

a. Reduce SP by 15%. The sales organization forecasts that at this


significantly reduced price, 2,100 units can be sold during the
remainder of the year. Total fixed costs and variable costs per unit
will stay as budgeted.

b. Lower VC per unit by $300 through the use of less-expensive direct


materials. The SP will also be reduced by $400, and sales of 1,750
units are expected for the remainder of the year.

c. Reduce fixed costs by 10% and lower the selling price by 30%. Variable
cost per unit will be unchanged. Sales of 2,200 units are expected for
the remainder of the year.

1. If no changes are made to the selling price or cost structure, determine


the number of units that Carlisle Engine Company must sell (a) to break
even and (b) to achieve its net income objective.

(a) Sales - TVC - TFC = 0


$4,000(X) - $1,000(X) - $4,800,000 = 0
$3,000(X) = $4,800,000
X = $4,800,000/$3,000 =
X= 1,600

(b) NI = IBT - Taxes


NI = IBT - (Tax Rate X IBT)
$1,200,000 = IBT - (.20)(IBT)
$1,200,000 = .80(IBT)
IBT = $1,200,000 / .80 = $1,500,000

Sales - TVC - TFC = IBT


Sales - TVC - TFC = $1,500,000
$4,000(X) - $1,000(X) - $4,800,000 = $1,500,000
$3,000(X) = $6,300,000
X = $6,300,000 / $3,000 = 2,100

2. Determine which alternative Carlise Engine should select to achieve the net
income objective.

Current situation: TCM = $3,000 X 400 = $1,200,000


TFC = 4,800,000
Unrecovered fixed costs ($3,600,000)

Alternative #a: Reduce the selling price by 15% to yield additional sales
of 2,100. $600 Change in selling price

Short way Longer way


Sales - TVC - TFC = IBT Sales - TVC - TFC = IBT
($4,000 - $600)(2,100) - $1,000(2,100) - $3,600,000 = IBT $4,000(400) + ($4,000 - $600)(
$3,400(2,100) - $1,000(2,100) - $3,600,000 = IBT $1,600,000 +$3,400(2,100
$2,400(2,100) - $3,600,000 = IBT $1,600,000 + $7,140,000 - $2,500,000 -
$5,040,000 - $3,600,000 = IBT $8,740,000 - $7,300,000 = IBT
IBT = $1,440,000 IBT =
NI = .80(IBT) $1,152,000 NI = .80(IBT)
Shortage ($48,000) Shortage
Net income objective of $1,200,000 is not achieved. Net income objective of $1,200,000 is not achieved

Alternative #b: Reduce the selling price by $400 and reduce


variable cost per unit by $300 to yield additional sales of 1,750.

Short way Longer way


Sales - TVC - TFC = IBT CM - TFC = IBT
($4,000-$400)(1,750) - ($1,000-$300)(1,750) - $3,600,000 = IBT $3,000(400) + [($4,000 - $400
$3,600(1,750) - $700(1,750) - $3,600,000 = IBT $1,200,000 + [$3,600(1,750
$2,900 (1,750) - $3,600,000 = IBT $1,200,000 + $2,900(1,750
$5,075,000 - $3,600,000 = IBT $1,200,000 +$5,075,000 -
IBT = $1,475,000 IBT =
NI = .80(IBT) $1,180,000 NI = .80(IBT)
Shortage ($20,000) Shortage
Net income objective of $1,200,000 is not achieved. Net income objective of $1,200,000 is not achieved

Alternative #c: Reduce the selling price by 30% and reduce


fixed cost by 10% to yield additional sales of 2,200.

New selling price = $4,000 - (.30)($4,000) = $4,000 - $1,200 = $2,800

Short way Longer way


Sales - TVC - TFC = IBT CM - TFC = IBT
$2,800(2,200) - $1,000(2,200) - ($3,600,000-(.10*$4,800,000)) = IBT $3,000(400) + $2,800(2,200)
$1,800(2,200) - ($3,600,000 - $480,000) = IBT $1,200,000 + $1,800(2,200
$3,960,000 - $3,120,000 = IBT $1,200,000 + $3,960,000 - $4,320,000 = IBT
IBT = $840,000 IBT =
NI = .80(IBT) $672,000 NI = .80(IBT)
Shortage ($528,000) Shortage
Net income objective of $1,200,000 is not achieved. Net income objective of $1,200,000 is not achieved

None of the alternatives will achieve the desired NI level of $1,200,000.


Selecting Alternative #b comes closest. Maybe it can be marginally modified
to make up the shortage of $20,000.
- TFC = IBT
+ ($4,000 - $600)(2,100) - $1,000(400+2,100) - $4,800,000 = IBT
+$3,400(2,100) - $1,000(400+2,100) - $4,800,000 = IBT
+ $7,140,000 - $2,500,000 - $4,800,000 = IBT
$7,300,000 = IBT
$1,440,000
$1,152,000
($48,000)
objective of $1,200,000 is not achieved.

+ [($4,000 - $400)(1,750) - ($1,000 - $300)(1,750)] - $4,800,000 = IBT


+ [$3,600(1,750) - $700(1,750)] - $4,800,000 = IBT
+ $2,900(1,750) - $4,800,000 = IBT
+$5,075,000 - $4,800,000 = IBT
$1,475,000
$1,180,000
($20,000)
objective of $1,200,000 is not achieved.

$2,800(2,200) - $1,000(2,200) - ($4,800,000-(.10*$4,800,000)) = IBT


+ $1,800(2,200) - ($4,800,000 - $480,000) = IBT
+ $3,960,000 - $4,320,000 = IBT
$840,000
$672,000
($528,000)
objective of $1,200,000 is not achieved.
Problem 3-44: Sales mix; three products

Given data:
The Ronowski Company has three product lines of belts - A, B, and C -
with contribution margins of $3, $2, and $1, respectively. The president
foresees sales of 200,000 units in the coming period, consisting of
20,000 units of A, 100,000 units of B, and 80,000 units of C. The
company's fixed costs for the period are $255,000.

Summary
Product CM/Unit Volume Mix TCM
A $3 20,000 10% $60,000
B $2 100,000 50% $200,000
C $1 80,000 40% $80,000
200,000 100% $340,000 $1.70

1. What is the company's breakeven point in units, assuming that the


given sales mix is maintained?

Sales = TVC + TFC + P


At breakeven: P = 0 and therefore
TCM = TFC
$3(.1)(X) + $2(.5)(X) + $1(.4)(X) = $255,000
$.3X + $1X + $.4(X) = $255,000
$1.7(X) = $255,000
X = $255,000/$1.7 = 150,000 Total units

Mix Total Units Units


A 10% 150,000 15,000
B 50% 150,000 75,000
C 40% 150,000 60,000
100% 150,000

2. If the sales mix is maintained, what is the total contribution margin


when 200,000 units are sold? What is operating income?

Mix Total Units Units CM/Unit TCM


A 10% 200,000 20,000 $3 $60,000 $0.30
B 50% 200,000 100,000 $2 $200,000 $1.00
C 40% 200,000 80,000 $1 $80,000 $0.40
100% 200,000 $340,000 $1.70
$340,000 $1.70
Operating income = TCM - Fixed Costs
OI = $340,000 - $255,000 $85,000
OI = $85,000

3. What would operating income be if 20,000 units of A, 80,000 units


of B, and 100,000 units of C were sold? What is the new BE point
in units if these relationships persist in the next period?
Mix Total Units Units CM/Unit TCM
A 10% 200,000 20,000 $3 $60,000
B 40% 200,000 80,000 $2 $160,000
C 50% 200,000 100,000 $1 $100,000
100% 200,000 $320,000 $1.60
$320,000
Operating income = TCM - Fixed Costs
OI = $320,000 - $255,000
OI = $65,000

Sales = TVC + TFC + P


At breakeven: P = 0 and therefore
TCM = TFC
$3(.1)(X) + $2(.4)(X) + $1(.5)(X) = $255,000
$.3X + $.8X + $.5(X) = $255,000
$1.6(X) = $255,000
X = $255,000/$1.6 = 159,375.00 Total units

Mix Total Units Units CM/Unit TCM


A 10% 159,375 15,937.50 $3 $47,812.50
B 40% 159,375 63,750.00 $2 $127,500.00
C 50% 159,375 79,687.50 $1 $79,687.50
100% 159,375.00 $255,000.00
Answer Proof
Problem 3-32: Uncertainty and expected costs.

Given data:
Hillmart Corp., an international retail giant, is considering implementing a new business to business (B2B) information syste
processing purchase orders. The current system costs Hillmart $1,000,000 per month and $45 per order. Hillmart has two
a partially automated B2B and a fully automated B2B system. The partially automated B2B system will have a fixed cost o
per month and a variable cost of $35 per order. The fully automated B2B system has a fixed cost of $11,000,000
per order.
Monthly Variable Costs
Summary: Fixed Costs Per Order
Current system $1,000,000 $45
Partially automated B2B system $5,000,000 $35
Fully Automated B2B system $11,000,000 $20

Based on data from the last two years, Hillmart has determined the following distribution of monthly orders:

Monthly # Expected
of Orders Probability # of Orders
300,000 0.15 45,000
400,000 0.20 80,000
500,000 0.40 200,000
600,000 0.15 90,000
700,000 0.10 70,000
1.00 485,000

1. Prepare a table showing the cost of each plan for each quantity of monthly orders.

Possible Number of monthly Orders


Current System: 300,000 400,000 500,000 600,000
Fixed amount $1,000,000 $1,000,000 $1,000,000 $1,000,000
Variable amount 13,500,000 18,000,000 22,500,000 27,000,000
Total anticipated cost $14,500,000 $19,000,000 $23,500,000 $28,000,000

Partially Automated B2B System: 300,000 400,000 500,000 600,000


Fixed amount $5,000,000 $5,000,000 $5,000,000 $5,000,000
Variable amount 10,500,000 14,000,000 17,500,000 21,000,000
Total anticipated cost $15,500,000 $19,000,000 $22,500,000 $26,000,000

Fully Automated B2B System: 300,000 400,000 500,000 600,000


Fixed amount $11,000,000 $11,000,000 $11,000,000 $11,000,000
Variable amount 6,000,000 8,000,000 10,000,000 12,000,000
Total anticipated cost $17,000,000 $19,000,000 $21,000,000 $23,000,000
Probability 0.15 0.20 0.40 0.15

2. What is the expected cost of each plan?


Short Way Long Way
Current System: $22,825,000 $22,825,000

Partially Automated B2B System: $21,975,000 $21,975,000


Fully Automated B2B System: $20,700,000 $20,700,000

3. In addition to the information systems costs, what other factors should Hillmart consider before deciding to implem
a new B2B system?

Hillmart should consider the impact of the different systems on its relationships with suppliers:

The interface with Hillmart's system may require that suppliers also update their systems. This could cause some suppl
the cost of their merchandise, or

It could force some suppliers to drop out of Hillmart's supply chain because the cost of the system change would be pro

Hillmart may also want to consider the reliability of the different systems.

Hillmart may also want to consider the effect on employee morale if employees have to be laid off as Hillmart
automates its systems.
siness (B2B) information system for
45 per order. Hillmart has two options,
ystem will have a fixed cost of $5,000,000
cost of $11,000,000 per month and $20

onthly orders:

Orders
700,000
$1,000,000
31,500,000
$32,500,000

700,000
$5,000,000
24,500,000
$29,500,000

700,000
$11,000,000
14,000,000
$25,000,000
0.10
er before deciding to implement

This could cause some suppliers to raise

system change would be prohibitive.

o be laid off as Hillmart


Problem 3-40: Alternate cost structures, uncertainty, and sensitivity analysis.

Given data:
Stylewise Printing Company currently leases its only copy machine for $1,000 a month. The
company is considering replacing this leasing agreement with a new contract that is entirely
commission based. Under the new agreement Stylewise would pay a commission for its printing
at a rate of $10 for every 500 pages printed. The company currently charges $0.15 per page to
its customers. The paper used in printing costs the company $.03 per page and other variable
costs, including hourly labor amounts to $.04 per page.

1. What is the company's breakeven point in units under the current leasing agreement?
What is it under the new commission based agreement?

Current leasing agreement -- fixed cost of $1,000 per month $1,000


Sales = TVC + TFC
$.15(X) = ($.03 + $.04)(X) + $1,000
$.08(X) = $1,000
X = $1,000/$.08 = 12,500 pages per month

New commission based agreement -- $10 per 500 pages, $10/500 = $.02 per page
Sales = TVC + TFC
$.15(X) = ($.03 + $.04 + $.02)(X) + $0
$.06(X) = $0
X = $0/$.06 = 0 pages per month

Conceptually superior solution:


What if comission is a true step fixed cost of $10 per step?
TCM = TFC B/E in pages Range
.08X = $10 125 1 500
.08X = $20 250 501 1,000
.08X = $30 375 1001 1,500
.08X = $40 500 1501 2,000
BE depends on the number of copies.
Every step increase of $10 (500 copies) requires an additional 125 pages to break even.
Note: In this case BE units is essentially 125. Why?

2. For what range of sales levels will Stylewise prefer


(a) the current fixed lease agreement
(b) the new commission agreement

Calculate the point of indifference:


Lease agreement -- fixed cost of $1,000 per month
Sales -TVC - TFC = P1
$.15(X) - ($.03 + $.04)(X) - $1,000 = P1
$.08X - $1,000 = P1

Commission based agreement -- $10 per 500 pages, $10/500 = $.02 per page
Sales -TVC - TFC = P2
$.15(X) - ($.03 + $.04 + $.02)(X) - $0 = P2
$.15X - $.09X - $0 = P2
$.06X = P2

At the point of indifference P1 = P2, therefore


Lease agreement: $.08X - $1,000 = P
Commission agreement: $.06X - 0 =P
$.08X - $1,000 = $.06X - 0
$.02X -$1,000 = 0
$.02X = $1,000
X = $1,000/$.02 = 50,000 pages/month

Thus Stylewise would prefer commission agreement from 0 to 50,000 pages.


and lease agreement above 50,000. Stylewise is indifferent at 50,000 units.

Conceptually superior solution:


What if comission is a true step fixed cost of $10 for every 500 pages printed?
Outside Range Range of Indifference is 49,501 - 50,000
Units Sold 49,500 49,501 49,502 49,999 50,000
Steps required= 99 100 100 100 100
Maximum Units 49,500 50,000 50,000 50,000 50,000
Profit
Lease Option $2,960.00 $2,960.08 $2,960.16 $2,999.92 $3,000.00
Commission Opt. $2,970.00 $2,960.08 $2,960.16 $2,999.92 $3,000.00
Difference $10 $0 $0 $0 $0
Preferred Option Commission Indifferent Indifferent Indifferent Indifferent
Preferred Option < than 49,501 49,501 to 50,000

3. Stylewise estimates that the company is equally likely to sell 20,000; 40,000; 60,000;
80,000; or 100,000 pages of print. Prepare a table that shows the expected profit at
each sales level under the lease agreement and under the commission agreement.
What is the expected value of each agreement? Which agreement should Stylewise
choose?

$.08X - $1,000 $.06X


Possible Most Likely Expected Profit from Profit from
Sales Amt. Probability Sales Leasing Commission
20,000 0.20 4,000 $120 $240
40,000 0.20 8,000 $440 $480
60,000 0.20 12,000 $760 $720
80,000 0.20 16,000 $1,080 $960
100,000 0.20 20,000 $1,400 $1,200
1.00 60,000 $3,800 $3,600 Long way
$3,800 $3,600 Easy way

The lease agreement is preferred since the expected profit is greater.


Alternatively, since the expected sales is greater than 50,000, the lease agreement is preferred.
ed?
Outside Range
50,001
101
50,500

$3,000.08
$2,990.08
$10
Lease
> than 50,000
Problem 3-22

Given data:
Fixed costs of $300,000
Variable costs % of 80%
Net income earned in 2002 was $84,000
Income tax rate is 40%

1. Compute the operating income.

NI = OI - Taxes
Taxes = Tax Rate X OI
NI = OI - (Taxes Rate X OI)
$84,000 = OI - (.4 X OI)
$84,000 = .6 X OI
OI = $84,000/.6 = $140,000

2. Compute the contribution margin.

Sales - TVC - TFC = OI


Sales - TVC = TFC + OI
TCM = $300,000 + $140,000 = $440,000

3. Compute the total sales revenue.


TCM = C/M % X Sales
C/M % = 1- V/C%
C/M % = 1- .80
C/M % = .20 = 20%
TCM = C/M% X Sales
$440,000 = .2 X Sales
Sales = $440,000 /.2 =
Sales = $2,200,000

4. Compute the breakeven sales revenue.

B/E = FC / C/M %

FC = $300,000 / .2 = $1,500,000
Problem 3-29

Given:
Annual athletic budget $5,000,000
Size of an individual athletic scholarship $20,000
Annual operating costs
Fixed portion $600,000
Variable portion per scholarship $2,000

1. Determine the number of athletic scholarships Midwest University


can offer each year.

Annual athletic budget $5,000,000


Operating Costs -- Fixed portion 600,000
Annual scholarship money available $4,400,000
Cost to the University per scholarship
Variable operating costs per scholarship $2,000
Scholarship amount 20,000 22,000
Maximum number of scholarships possible 200

2. Suppose that the total budget for next year is reduced by 22%.
Fixed costs are to remain the same. Calculate the number of
athletic scholarships that Midwest can offer next year.

Annual athletic budget $5,000,000


Less: 22% budget reduction (1,100,000)
Adjusted budget for next year $3,900,000
Operating Costs -- Fixed portion 600,000
Annual scholarship money available $3,300,000
Cost to the University per scholarship
Variable operating costs per scholarship $2,000
Scholarship amount 20,000 22,000
Maximum number of scholarships possible 150

3. Suppose that the total budget for next year is reduced by 22%.
Fixed costs are to remain the same. If Midwest wanted to offer
the same number of scholarships as it did in requirement 1,
calculate the amount that will be paid to each scholarship student.

Adjusted budget for next year $3,900,000


Operating Costs -- Fixed portion 600,000
Annual scholarship money available $3,300,000
Maximum number of scholarships possible 200
Cost to the University per scholarship $16,500
Variable operating costs per scholarship (2,000)
Scholarship amount distributable per scholarship student $14,500
Problem 3-37

Given data:

Annual Capacity
Currently available 50,000
Currently used 40,000

Selling price expected per radio = $105


Variable cost per radio:
Manufacturing $45
Marketing & Distribution 10 $55
Fixed costs:
Manufacturing $800,000
Marketing & Distribution 600,000 $1,400,000

1. The Marketing Department indicates that decreasing the selling


price to $99 would increase sales to 50,000 units. This strategy
would require Tocchet to increase its fixed marketing and
distribution costs. Calculate the maximum increase in fixed
M & D costs that will allow Tocchet to reduce the SP to $99 and
maintain its operating income.

Current operating income:

Sales - TVC - TFC = P


$105(40,000) - $55(40,000) - $1,400,000 = P
P= $600,000

New situation:
Let X = the maximum increase in marketing & distribution costs
Sales - TVC - TFC = P
$99(50,000) - $55(50,000) - $1,400,000 - X = $600,000
$44(50,000) - $2,000,000 = X
X= $200,000

2. The Manufacturing Department proposes changes in the


manufacturing process to add new features to the CB1 radio
These changes will increase fixed manufacturing costs by
$100,000 and variable manufacturing cost per unit by $2.
At its current sales level of 40,000 units, compute the minimum
selling price that will allow Tocchet to add these new features
and maintain its operating income.

Sales - TVC - TFC = P


(X)(40,000) - $57(40,000) - $1,500,000 = $600,000
40,000(X) - $2,280,000 - $1,500,000 = $600,000
40,000 (X) = $2,280,000 + $1,500,000 + $600,000
X = $4,380,000 / 40,000
X= $109.50
Problem 3-45

Given data:
Evenkeel Corporation manufactures and sells one product - an infant car seat
called Plumar - at a price of $50. Variable costs equal $20 per car seat. Fixed
costs are $495,000. Evenkeel manufactures Plumar upon the receipt of orders
from its customers. In 2005, it sold 30,000 units of Plumar. One of Evenkeel's
customers, Glaston Corporation, has asked if in 2006 Evenkeel will produce
a different style of car seat called Ridex. Glaston will pay $25 for each unit of
Ridex. The variable cost for Ridex is estimated to be $15 per seat. Evenkeel
has enough capacity to manufacture all the units of Plumar it can sell as well as
the units of Ridex that Glaston wants without incurring any additional fixed
costs. Evenkeel estimates that in 2006 it will sell 30,000 units of Plumar
(assuming the same SP and VC as in 2005) and 20,000 units of Ridex.

Andy Minton, the president of Evenkeel, checks the effect of accepting Glaston's
offer on the BE revenues for 2006. Using the planned sales mix for 2006, he is
surprised to find that the revenues required to break even appear to increase.
he is not sure that his numbers are correct, but if they are, Andy feels inclined to
reject Glaston's offer. He asks for your advice.

Summary of Data:
Selling price per Plumar (infant car seat) = $50
Variable cost per Plumar = $20
Annual fixed costs = $495,000
Expected sales volume of Plumar = 30,000
Selling price per Ridex = $25
Variable cost per Ridex = $15
Expected sales volume of Ridex = 20,000
Plenty of excess capacity to sell both products

1. Determine the breakeven point in units and dollars for 2005.

Sales - TVC - TFC = IBT = 0


Let X = the number of units sold to breakeven.
$50(X) - $20(X) - $495,000 = 0
$30(X) = $495,000
X = $495,000 / $30 = 16,500 Units of Plumar
Sales = 16,500 X $50 = $825,000

2. Determine the breakeven point in units and dollars for 2006 at


the planned sales mix (30,000 units of Plumar and 20,000 units
Ridex).
Sales - TVC - TFC = IBT = 0
$50(P) - $20(P) + $25(R) - $15(R) - $495,000 = 0
$30(P) + $10(R) - $495,000 = 0
Let X = the total number of units sold.
$30(3/5)(X) + $10(2/5)(X) - $495,000 = 0
($90/5)(X) + ($20/5)(X) - $495,000 = 0
$18X + $4X - $495,000 = 0
$22X - $495,000 = 0
$22X = $495,000
X= 22,500 Total units

Units Sales C/M C/M %


Plumar = 3/5(X) = 13,500 $675,000 $405,000 60%
Ridex = 2/5(X)= 9,000 225,000 90,000 40%
22,500 $900,000 $495,000 55%

3. Explain why the B/E points in revenues calculated in requirement 1


and requirement 2 are different.
Sales mix differs.

Sales Mix
Requirement #
Product 1 2 CM/Unit
Plumar 100% 60% $30
Ridex 0% 40% $10

In requirement 2 less sales from higher C/M generator, so more sales are
needed to generate $495,000 of TCM.

Req. #1 CM/Unit TCM


Plumar 13,500 $30 $405,000
Plumar 3,000 $30 $90,000
Total 16,500 $30 $495,000

Req. #2 CM/Unit TCM


Plumar 13,500 $30 $405,000
Ridex 9,000 $10 $90,000
Total 22,500 $22 $495,000

In requirement 2 less sales from higher C/M % generator, so more total sales
dollars are needed to generate $495,000 of TCM.

Req. #1 C/M% TCM


Plumar $675,000 60% $405,000
Plumar 150,000 60% $90,000
Total $825,000 60% $495,000

Req. #2 C/M% TCM


Plumar $675,000 60% $405,000
Ridex 225,000 40% $90,000
Total $900,000 55% $495,000

4. Should both products be sold?

Only Plumar:
$30(30,000) -$495,000 = IBT = $405,000

Both:
Plumar $30(30,000) -$495,000 = IBT = $405,000
Ridex $10(20,000) - 0 = IBT = 200,000
$605,000
Sell both.

S-ar putea să vă placă și